Harmonic series partial sum











up vote
0
down vote

favorite
1












According to Beyer (1987) the following progression is called harmonic series: $$frac {1} {a_{1}},frac {1} {a_{1}+d}, frac {1} {a_{1}+2d},...$$



How it can be calculated the partial sum of the above mentioned sequence?










share|cite|improve this question






















  • I don´t see that there exists a closed form for the partial sum.
    – callculus
    Nov 17 at 16:03










  • @callculus what about approximation?
    – David
    Nov 17 at 16:03






  • 1




    Search for "Harmonic numbers". Note however, that your sequence is more general, related to "Digamma function"
    – Yuriy S
    Nov 17 at 16:06










  • @YuriyS Thank you for your comment. Can you please provide some material reagrdinf this problem?
    – David
    Nov 17 at 16:10















up vote
0
down vote

favorite
1












According to Beyer (1987) the following progression is called harmonic series: $$frac {1} {a_{1}},frac {1} {a_{1}+d}, frac {1} {a_{1}+2d},...$$



How it can be calculated the partial sum of the above mentioned sequence?










share|cite|improve this question






















  • I don´t see that there exists a closed form for the partial sum.
    – callculus
    Nov 17 at 16:03










  • @callculus what about approximation?
    – David
    Nov 17 at 16:03






  • 1




    Search for "Harmonic numbers". Note however, that your sequence is more general, related to "Digamma function"
    – Yuriy S
    Nov 17 at 16:06










  • @YuriyS Thank you for your comment. Can you please provide some material reagrdinf this problem?
    – David
    Nov 17 at 16:10













up vote
0
down vote

favorite
1









up vote
0
down vote

favorite
1






1





According to Beyer (1987) the following progression is called harmonic series: $$frac {1} {a_{1}},frac {1} {a_{1}+d}, frac {1} {a_{1}+2d},...$$



How it can be calculated the partial sum of the above mentioned sequence?










share|cite|improve this question













According to Beyer (1987) the following progression is called harmonic series: $$frac {1} {a_{1}},frac {1} {a_{1}+d}, frac {1} {a_{1}+2d},...$$



How it can be calculated the partial sum of the above mentioned sequence?







calculus sequences-and-series






share|cite|improve this question













share|cite|improve this question











share|cite|improve this question




share|cite|improve this question










asked Nov 17 at 15:58









David

408




408












  • I don´t see that there exists a closed form for the partial sum.
    – callculus
    Nov 17 at 16:03










  • @callculus what about approximation?
    – David
    Nov 17 at 16:03






  • 1




    Search for "Harmonic numbers". Note however, that your sequence is more general, related to "Digamma function"
    – Yuriy S
    Nov 17 at 16:06










  • @YuriyS Thank you for your comment. Can you please provide some material reagrdinf this problem?
    – David
    Nov 17 at 16:10


















  • I don´t see that there exists a closed form for the partial sum.
    – callculus
    Nov 17 at 16:03










  • @callculus what about approximation?
    – David
    Nov 17 at 16:03






  • 1




    Search for "Harmonic numbers". Note however, that your sequence is more general, related to "Digamma function"
    – Yuriy S
    Nov 17 at 16:06










  • @YuriyS Thank you for your comment. Can you please provide some material reagrdinf this problem?
    – David
    Nov 17 at 16:10
















I don´t see that there exists a closed form for the partial sum.
– callculus
Nov 17 at 16:03




I don´t see that there exists a closed form for the partial sum.
– callculus
Nov 17 at 16:03












@callculus what about approximation?
– David
Nov 17 at 16:03




@callculus what about approximation?
– David
Nov 17 at 16:03




1




1




Search for "Harmonic numbers". Note however, that your sequence is more general, related to "Digamma function"
– Yuriy S
Nov 17 at 16:06




Search for "Harmonic numbers". Note however, that your sequence is more general, related to "Digamma function"
– Yuriy S
Nov 17 at 16:06












@YuriyS Thank you for your comment. Can you please provide some material reagrdinf this problem?
– David
Nov 17 at 16:10




@YuriyS Thank you for your comment. Can you please provide some material reagrdinf this problem?
– David
Nov 17 at 16:10










3 Answers
3






active

oldest

votes

















up vote
3
down vote



accepted










The sum of terms for $n=0,1,...,m$ is $$sum_{n=0}^m frac{1}{d(a_1/d+n)} =frac{1}{d} left(psi left(m+1+frac{a_1}{d}right) -psi left(frac{a_1}{d}right) right)$$



Where $psi$ is the Digamma function.



Use the definition:



$$psi(z)=-gamma+sum_{n=0}^infty frac{z-1}{(n+1)(n+z)}$$






share|cite|improve this answer























  • Thank you for your answer. Could you please provide some information how it is derived? P.S. sorry, if my comment is weird, i don't have background in this topic.
    – David
    Nov 17 at 16:23










  • @David, try using the definition I provided, and you will see that the formula is true
    – Yuriy S
    Nov 17 at 16:25










  • Thank you. My aim is to find a closed-form expression for the finite series. Is it possible?
    – David
    Nov 17 at 16:31










  • @David, I have written it in my answer above
    – Yuriy S
    Nov 17 at 16:38










  • The Digamma function is expressed in terms of infinite sum, but my aim is to find closed-form expression. Thank you.
    – David
    Nov 17 at 16:52


















up vote
2
down vote













Yuriy S gave the only possible closed form for the summation.



For large $n$, for sure, you could use series expansions and get from
$$S_n=sum_{i=0}^n frac{1}{a+i,d}=frac 1d left(psi left(n+1+frac{a}{d}right)-psi left(frac{a}{d}right) right)$$
$$S_n=frac{log(n)-psi left(frac{a}{d}right)}{d}+frac{2 a+d}{2 d^2 n}-frac{6 a^2+6 a d+d^2}{12
d^3 n^2}+frac{2 a^3+3 a^2 d+a d^2}{6 d^4
n^3}+Oleft(frac{1}{n^4}right)$$



Using $a=pi$, $d=e$ and $n=10$, the exact result would be $approx 1.03107$ while the above series expansion would give $approx 1.03113$.






share|cite|improve this answer





















  • Please have a look. Thank you for your support. math.stackexchange.com/questions/3003278/…
    – David
    Nov 18 at 8:56










  • @David. No idea about this other one. I am quite skeptical about even an approximation. Even the case $a=alpha=0$ makes the problem more than difficult (at least to me).
    – Claude Leibovici
    Nov 18 at 9:11










  • Do you mean both of them?
    – David
    Nov 18 at 9:12


















up vote
1
down vote













The Harmonic Series



The partial sum of the standard Harmonic Series is given by
$$
H_n=sum_{k=1}^nfrac1ktag1
$$

This can be extended to a function that is analytic except at the negative integers
$$
H(z)=sum_{k=1}^inftyleft(frac1k-frac1{k+z}right)tag2
$$

The Euler-Maclaurin Sum Formula gives the asymptotic expansion
$$
H_n=log(n)+gamma+frac1{2n}-frac1{12n^2}+frac1{120n^4}-frac1{252n^6}+frac1{240n^8}-frac1{132n^{10}}+O!left(frac1{n^{12}}right)tag3
$$

where $gamma=0.57721566490153286060651209$ is the Euler-Mascheroni Constant.





The Series in the Question
$$
begin{align}
sum_{k=1}^nfrac1{a+(k-1)d}
&=frac1dsum_{k=1}^nfrac1{frac ad+k-1}\
&=frac1dsum_{k=1}^inftyleft(frac1{frac ad+k-1}-frac1{frac{a-d}d+k-1+n}right)\[3pt]
&=frac1dleft(H!left(n+frac ad-1right)-H!left(frac ad-1right)right)tag4
end{align}
$$

using the extension in $(2)$.



Formula $(2)$ from this answer allows us to compute $H!left(frac ad-1right)$ for rational $a$ and $d$, while formula $(3)$ above allows us to approximate $H!left(n+frac ad-1right)$ for large $n$.






share|cite|improve this answer























  • Thank you for your answer.
    – David
    Nov 18 at 20:34










  • Please have a look. math.stackexchange.com/questions/3003278/…
    – David
    Nov 18 at 20:35










  • Could you, please, support in order to calculate or approximate the sums represented in the above mentioned link?
    – David
    Nov 18 at 20:36











Your Answer





StackExchange.ifUsing("editor", function () {
return StackExchange.using("mathjaxEditing", function () {
StackExchange.MarkdownEditor.creationCallbacks.add(function (editor, postfix) {
StackExchange.mathjaxEditing.prepareWmdForMathJax(editor, postfix, [["$", "$"], ["\\(","\\)"]]);
});
});
}, "mathjax-editing");

StackExchange.ready(function() {
var channelOptions = {
tags: "".split(" "),
id: "69"
};
initTagRenderer("".split(" "), "".split(" "), channelOptions);

StackExchange.using("externalEditor", function() {
// Have to fire editor after snippets, if snippets enabled
if (StackExchange.settings.snippets.snippetsEnabled) {
StackExchange.using("snippets", function() {
createEditor();
});
}
else {
createEditor();
}
});

function createEditor() {
StackExchange.prepareEditor({
heartbeatType: 'answer',
convertImagesToLinks: true,
noModals: true,
showLowRepImageUploadWarning: true,
reputationToPostImages: 10,
bindNavPrevention: true,
postfix: "",
imageUploader: {
brandingHtml: "Powered by u003ca class="icon-imgur-white" href="https://imgur.com/"u003eu003c/au003e",
contentPolicyHtml: "User contributions licensed under u003ca href="https://creativecommons.org/licenses/by-sa/3.0/"u003ecc by-sa 3.0 with attribution requiredu003c/au003e u003ca href="https://stackoverflow.com/legal/content-policy"u003e(content policy)u003c/au003e",
allowUrls: true
},
noCode: true, onDemand: true,
discardSelector: ".discard-answer"
,immediatelyShowMarkdownHelp:true
});


}
});














draft saved

draft discarded


















StackExchange.ready(
function () {
StackExchange.openid.initPostLogin('.new-post-login', 'https%3a%2f%2fmath.stackexchange.com%2fquestions%2f3002502%2fharmonic-series-partial-sum%23new-answer', 'question_page');
}
);

Post as a guest















Required, but never shown

























3 Answers
3






active

oldest

votes








3 Answers
3






active

oldest

votes









active

oldest

votes






active

oldest

votes








up vote
3
down vote



accepted










The sum of terms for $n=0,1,...,m$ is $$sum_{n=0}^m frac{1}{d(a_1/d+n)} =frac{1}{d} left(psi left(m+1+frac{a_1}{d}right) -psi left(frac{a_1}{d}right) right)$$



Where $psi$ is the Digamma function.



Use the definition:



$$psi(z)=-gamma+sum_{n=0}^infty frac{z-1}{(n+1)(n+z)}$$






share|cite|improve this answer























  • Thank you for your answer. Could you please provide some information how it is derived? P.S. sorry, if my comment is weird, i don't have background in this topic.
    – David
    Nov 17 at 16:23










  • @David, try using the definition I provided, and you will see that the formula is true
    – Yuriy S
    Nov 17 at 16:25










  • Thank you. My aim is to find a closed-form expression for the finite series. Is it possible?
    – David
    Nov 17 at 16:31










  • @David, I have written it in my answer above
    – Yuriy S
    Nov 17 at 16:38










  • The Digamma function is expressed in terms of infinite sum, but my aim is to find closed-form expression. Thank you.
    – David
    Nov 17 at 16:52















up vote
3
down vote



accepted










The sum of terms for $n=0,1,...,m$ is $$sum_{n=0}^m frac{1}{d(a_1/d+n)} =frac{1}{d} left(psi left(m+1+frac{a_1}{d}right) -psi left(frac{a_1}{d}right) right)$$



Where $psi$ is the Digamma function.



Use the definition:



$$psi(z)=-gamma+sum_{n=0}^infty frac{z-1}{(n+1)(n+z)}$$






share|cite|improve this answer























  • Thank you for your answer. Could you please provide some information how it is derived? P.S. sorry, if my comment is weird, i don't have background in this topic.
    – David
    Nov 17 at 16:23










  • @David, try using the definition I provided, and you will see that the formula is true
    – Yuriy S
    Nov 17 at 16:25










  • Thank you. My aim is to find a closed-form expression for the finite series. Is it possible?
    – David
    Nov 17 at 16:31










  • @David, I have written it in my answer above
    – Yuriy S
    Nov 17 at 16:38










  • The Digamma function is expressed in terms of infinite sum, but my aim is to find closed-form expression. Thank you.
    – David
    Nov 17 at 16:52













up vote
3
down vote



accepted







up vote
3
down vote



accepted






The sum of terms for $n=0,1,...,m$ is $$sum_{n=0}^m frac{1}{d(a_1/d+n)} =frac{1}{d} left(psi left(m+1+frac{a_1}{d}right) -psi left(frac{a_1}{d}right) right)$$



Where $psi$ is the Digamma function.



Use the definition:



$$psi(z)=-gamma+sum_{n=0}^infty frac{z-1}{(n+1)(n+z)}$$






share|cite|improve this answer














The sum of terms for $n=0,1,...,m$ is $$sum_{n=0}^m frac{1}{d(a_1/d+n)} =frac{1}{d} left(psi left(m+1+frac{a_1}{d}right) -psi left(frac{a_1}{d}right) right)$$



Where $psi$ is the Digamma function.



Use the definition:



$$psi(z)=-gamma+sum_{n=0}^infty frac{z-1}{(n+1)(n+z)}$$







share|cite|improve this answer














share|cite|improve this answer



share|cite|improve this answer








edited Nov 17 at 16:23

























answered Nov 17 at 16:16









Yuriy S

15.3k433115




15.3k433115












  • Thank you for your answer. Could you please provide some information how it is derived? P.S. sorry, if my comment is weird, i don't have background in this topic.
    – David
    Nov 17 at 16:23










  • @David, try using the definition I provided, and you will see that the formula is true
    – Yuriy S
    Nov 17 at 16:25










  • Thank you. My aim is to find a closed-form expression for the finite series. Is it possible?
    – David
    Nov 17 at 16:31










  • @David, I have written it in my answer above
    – Yuriy S
    Nov 17 at 16:38










  • The Digamma function is expressed in terms of infinite sum, but my aim is to find closed-form expression. Thank you.
    – David
    Nov 17 at 16:52


















  • Thank you for your answer. Could you please provide some information how it is derived? P.S. sorry, if my comment is weird, i don't have background in this topic.
    – David
    Nov 17 at 16:23










  • @David, try using the definition I provided, and you will see that the formula is true
    – Yuriy S
    Nov 17 at 16:25










  • Thank you. My aim is to find a closed-form expression for the finite series. Is it possible?
    – David
    Nov 17 at 16:31










  • @David, I have written it in my answer above
    – Yuriy S
    Nov 17 at 16:38










  • The Digamma function is expressed in terms of infinite sum, but my aim is to find closed-form expression. Thank you.
    – David
    Nov 17 at 16:52
















Thank you for your answer. Could you please provide some information how it is derived? P.S. sorry, if my comment is weird, i don't have background in this topic.
– David
Nov 17 at 16:23




Thank you for your answer. Could you please provide some information how it is derived? P.S. sorry, if my comment is weird, i don't have background in this topic.
– David
Nov 17 at 16:23












@David, try using the definition I provided, and you will see that the formula is true
– Yuriy S
Nov 17 at 16:25




@David, try using the definition I provided, and you will see that the formula is true
– Yuriy S
Nov 17 at 16:25












Thank you. My aim is to find a closed-form expression for the finite series. Is it possible?
– David
Nov 17 at 16:31




Thank you. My aim is to find a closed-form expression for the finite series. Is it possible?
– David
Nov 17 at 16:31












@David, I have written it in my answer above
– Yuriy S
Nov 17 at 16:38




@David, I have written it in my answer above
– Yuriy S
Nov 17 at 16:38












The Digamma function is expressed in terms of infinite sum, but my aim is to find closed-form expression. Thank you.
– David
Nov 17 at 16:52




The Digamma function is expressed in terms of infinite sum, but my aim is to find closed-form expression. Thank you.
– David
Nov 17 at 16:52










up vote
2
down vote













Yuriy S gave the only possible closed form for the summation.



For large $n$, for sure, you could use series expansions and get from
$$S_n=sum_{i=0}^n frac{1}{a+i,d}=frac 1d left(psi left(n+1+frac{a}{d}right)-psi left(frac{a}{d}right) right)$$
$$S_n=frac{log(n)-psi left(frac{a}{d}right)}{d}+frac{2 a+d}{2 d^2 n}-frac{6 a^2+6 a d+d^2}{12
d^3 n^2}+frac{2 a^3+3 a^2 d+a d^2}{6 d^4
n^3}+Oleft(frac{1}{n^4}right)$$



Using $a=pi$, $d=e$ and $n=10$, the exact result would be $approx 1.03107$ while the above series expansion would give $approx 1.03113$.






share|cite|improve this answer





















  • Please have a look. Thank you for your support. math.stackexchange.com/questions/3003278/…
    – David
    Nov 18 at 8:56










  • @David. No idea about this other one. I am quite skeptical about even an approximation. Even the case $a=alpha=0$ makes the problem more than difficult (at least to me).
    – Claude Leibovici
    Nov 18 at 9:11










  • Do you mean both of them?
    – David
    Nov 18 at 9:12















up vote
2
down vote













Yuriy S gave the only possible closed form for the summation.



For large $n$, for sure, you could use series expansions and get from
$$S_n=sum_{i=0}^n frac{1}{a+i,d}=frac 1d left(psi left(n+1+frac{a}{d}right)-psi left(frac{a}{d}right) right)$$
$$S_n=frac{log(n)-psi left(frac{a}{d}right)}{d}+frac{2 a+d}{2 d^2 n}-frac{6 a^2+6 a d+d^2}{12
d^3 n^2}+frac{2 a^3+3 a^2 d+a d^2}{6 d^4
n^3}+Oleft(frac{1}{n^4}right)$$



Using $a=pi$, $d=e$ and $n=10$, the exact result would be $approx 1.03107$ while the above series expansion would give $approx 1.03113$.






share|cite|improve this answer





















  • Please have a look. Thank you for your support. math.stackexchange.com/questions/3003278/…
    – David
    Nov 18 at 8:56










  • @David. No idea about this other one. I am quite skeptical about even an approximation. Even the case $a=alpha=0$ makes the problem more than difficult (at least to me).
    – Claude Leibovici
    Nov 18 at 9:11










  • Do you mean both of them?
    – David
    Nov 18 at 9:12













up vote
2
down vote










up vote
2
down vote









Yuriy S gave the only possible closed form for the summation.



For large $n$, for sure, you could use series expansions and get from
$$S_n=sum_{i=0}^n frac{1}{a+i,d}=frac 1d left(psi left(n+1+frac{a}{d}right)-psi left(frac{a}{d}right) right)$$
$$S_n=frac{log(n)-psi left(frac{a}{d}right)}{d}+frac{2 a+d}{2 d^2 n}-frac{6 a^2+6 a d+d^2}{12
d^3 n^2}+frac{2 a^3+3 a^2 d+a d^2}{6 d^4
n^3}+Oleft(frac{1}{n^4}right)$$



Using $a=pi$, $d=e$ and $n=10$, the exact result would be $approx 1.03107$ while the above series expansion would give $approx 1.03113$.






share|cite|improve this answer












Yuriy S gave the only possible closed form for the summation.



For large $n$, for sure, you could use series expansions and get from
$$S_n=sum_{i=0}^n frac{1}{a+i,d}=frac 1d left(psi left(n+1+frac{a}{d}right)-psi left(frac{a}{d}right) right)$$
$$S_n=frac{log(n)-psi left(frac{a}{d}right)}{d}+frac{2 a+d}{2 d^2 n}-frac{6 a^2+6 a d+d^2}{12
d^3 n^2}+frac{2 a^3+3 a^2 d+a d^2}{6 d^4
n^3}+Oleft(frac{1}{n^4}right)$$



Using $a=pi$, $d=e$ and $n=10$, the exact result would be $approx 1.03107$ while the above series expansion would give $approx 1.03113$.







share|cite|improve this answer












share|cite|improve this answer



share|cite|improve this answer










answered Nov 18 at 4:40









Claude Leibovici

117k1156131




117k1156131












  • Please have a look. Thank you for your support. math.stackexchange.com/questions/3003278/…
    – David
    Nov 18 at 8:56










  • @David. No idea about this other one. I am quite skeptical about even an approximation. Even the case $a=alpha=0$ makes the problem more than difficult (at least to me).
    – Claude Leibovici
    Nov 18 at 9:11










  • Do you mean both of them?
    – David
    Nov 18 at 9:12


















  • Please have a look. Thank you for your support. math.stackexchange.com/questions/3003278/…
    – David
    Nov 18 at 8:56










  • @David. No idea about this other one. I am quite skeptical about even an approximation. Even the case $a=alpha=0$ makes the problem more than difficult (at least to me).
    – Claude Leibovici
    Nov 18 at 9:11










  • Do you mean both of them?
    – David
    Nov 18 at 9:12
















Please have a look. Thank you for your support. math.stackexchange.com/questions/3003278/…
– David
Nov 18 at 8:56




Please have a look. Thank you for your support. math.stackexchange.com/questions/3003278/…
– David
Nov 18 at 8:56












@David. No idea about this other one. I am quite skeptical about even an approximation. Even the case $a=alpha=0$ makes the problem more than difficult (at least to me).
– Claude Leibovici
Nov 18 at 9:11




@David. No idea about this other one. I am quite skeptical about even an approximation. Even the case $a=alpha=0$ makes the problem more than difficult (at least to me).
– Claude Leibovici
Nov 18 at 9:11












Do you mean both of them?
– David
Nov 18 at 9:12




Do you mean both of them?
– David
Nov 18 at 9:12










up vote
1
down vote













The Harmonic Series



The partial sum of the standard Harmonic Series is given by
$$
H_n=sum_{k=1}^nfrac1ktag1
$$

This can be extended to a function that is analytic except at the negative integers
$$
H(z)=sum_{k=1}^inftyleft(frac1k-frac1{k+z}right)tag2
$$

The Euler-Maclaurin Sum Formula gives the asymptotic expansion
$$
H_n=log(n)+gamma+frac1{2n}-frac1{12n^2}+frac1{120n^4}-frac1{252n^6}+frac1{240n^8}-frac1{132n^{10}}+O!left(frac1{n^{12}}right)tag3
$$

where $gamma=0.57721566490153286060651209$ is the Euler-Mascheroni Constant.





The Series in the Question
$$
begin{align}
sum_{k=1}^nfrac1{a+(k-1)d}
&=frac1dsum_{k=1}^nfrac1{frac ad+k-1}\
&=frac1dsum_{k=1}^inftyleft(frac1{frac ad+k-1}-frac1{frac{a-d}d+k-1+n}right)\[3pt]
&=frac1dleft(H!left(n+frac ad-1right)-H!left(frac ad-1right)right)tag4
end{align}
$$

using the extension in $(2)$.



Formula $(2)$ from this answer allows us to compute $H!left(frac ad-1right)$ for rational $a$ and $d$, while formula $(3)$ above allows us to approximate $H!left(n+frac ad-1right)$ for large $n$.






share|cite|improve this answer























  • Thank you for your answer.
    – David
    Nov 18 at 20:34










  • Please have a look. math.stackexchange.com/questions/3003278/…
    – David
    Nov 18 at 20:35










  • Could you, please, support in order to calculate or approximate the sums represented in the above mentioned link?
    – David
    Nov 18 at 20:36















up vote
1
down vote













The Harmonic Series



The partial sum of the standard Harmonic Series is given by
$$
H_n=sum_{k=1}^nfrac1ktag1
$$

This can be extended to a function that is analytic except at the negative integers
$$
H(z)=sum_{k=1}^inftyleft(frac1k-frac1{k+z}right)tag2
$$

The Euler-Maclaurin Sum Formula gives the asymptotic expansion
$$
H_n=log(n)+gamma+frac1{2n}-frac1{12n^2}+frac1{120n^4}-frac1{252n^6}+frac1{240n^8}-frac1{132n^{10}}+O!left(frac1{n^{12}}right)tag3
$$

where $gamma=0.57721566490153286060651209$ is the Euler-Mascheroni Constant.





The Series in the Question
$$
begin{align}
sum_{k=1}^nfrac1{a+(k-1)d}
&=frac1dsum_{k=1}^nfrac1{frac ad+k-1}\
&=frac1dsum_{k=1}^inftyleft(frac1{frac ad+k-1}-frac1{frac{a-d}d+k-1+n}right)\[3pt]
&=frac1dleft(H!left(n+frac ad-1right)-H!left(frac ad-1right)right)tag4
end{align}
$$

using the extension in $(2)$.



Formula $(2)$ from this answer allows us to compute $H!left(frac ad-1right)$ for rational $a$ and $d$, while formula $(3)$ above allows us to approximate $H!left(n+frac ad-1right)$ for large $n$.






share|cite|improve this answer























  • Thank you for your answer.
    – David
    Nov 18 at 20:34










  • Please have a look. math.stackexchange.com/questions/3003278/…
    – David
    Nov 18 at 20:35










  • Could you, please, support in order to calculate or approximate the sums represented in the above mentioned link?
    – David
    Nov 18 at 20:36













up vote
1
down vote










up vote
1
down vote









The Harmonic Series



The partial sum of the standard Harmonic Series is given by
$$
H_n=sum_{k=1}^nfrac1ktag1
$$

This can be extended to a function that is analytic except at the negative integers
$$
H(z)=sum_{k=1}^inftyleft(frac1k-frac1{k+z}right)tag2
$$

The Euler-Maclaurin Sum Formula gives the asymptotic expansion
$$
H_n=log(n)+gamma+frac1{2n}-frac1{12n^2}+frac1{120n^4}-frac1{252n^6}+frac1{240n^8}-frac1{132n^{10}}+O!left(frac1{n^{12}}right)tag3
$$

where $gamma=0.57721566490153286060651209$ is the Euler-Mascheroni Constant.





The Series in the Question
$$
begin{align}
sum_{k=1}^nfrac1{a+(k-1)d}
&=frac1dsum_{k=1}^nfrac1{frac ad+k-1}\
&=frac1dsum_{k=1}^inftyleft(frac1{frac ad+k-1}-frac1{frac{a-d}d+k-1+n}right)\[3pt]
&=frac1dleft(H!left(n+frac ad-1right)-H!left(frac ad-1right)right)tag4
end{align}
$$

using the extension in $(2)$.



Formula $(2)$ from this answer allows us to compute $H!left(frac ad-1right)$ for rational $a$ and $d$, while formula $(3)$ above allows us to approximate $H!left(n+frac ad-1right)$ for large $n$.






share|cite|improve this answer














The Harmonic Series



The partial sum of the standard Harmonic Series is given by
$$
H_n=sum_{k=1}^nfrac1ktag1
$$

This can be extended to a function that is analytic except at the negative integers
$$
H(z)=sum_{k=1}^inftyleft(frac1k-frac1{k+z}right)tag2
$$

The Euler-Maclaurin Sum Formula gives the asymptotic expansion
$$
H_n=log(n)+gamma+frac1{2n}-frac1{12n^2}+frac1{120n^4}-frac1{252n^6}+frac1{240n^8}-frac1{132n^{10}}+O!left(frac1{n^{12}}right)tag3
$$

where $gamma=0.57721566490153286060651209$ is the Euler-Mascheroni Constant.





The Series in the Question
$$
begin{align}
sum_{k=1}^nfrac1{a+(k-1)d}
&=frac1dsum_{k=1}^nfrac1{frac ad+k-1}\
&=frac1dsum_{k=1}^inftyleft(frac1{frac ad+k-1}-frac1{frac{a-d}d+k-1+n}right)\[3pt]
&=frac1dleft(H!left(n+frac ad-1right)-H!left(frac ad-1right)right)tag4
end{align}
$$

using the extension in $(2)$.



Formula $(2)$ from this answer allows us to compute $H!left(frac ad-1right)$ for rational $a$ and $d$, while formula $(3)$ above allows us to approximate $H!left(n+frac ad-1right)$ for large $n$.







share|cite|improve this answer














share|cite|improve this answer



share|cite|improve this answer








edited Nov 18 at 17:42

























answered Nov 18 at 15:15









robjohn

263k27301623




263k27301623












  • Thank you for your answer.
    – David
    Nov 18 at 20:34










  • Please have a look. math.stackexchange.com/questions/3003278/…
    – David
    Nov 18 at 20:35










  • Could you, please, support in order to calculate or approximate the sums represented in the above mentioned link?
    – David
    Nov 18 at 20:36


















  • Thank you for your answer.
    – David
    Nov 18 at 20:34










  • Please have a look. math.stackexchange.com/questions/3003278/…
    – David
    Nov 18 at 20:35










  • Could you, please, support in order to calculate or approximate the sums represented in the above mentioned link?
    – David
    Nov 18 at 20:36
















Thank you for your answer.
– David
Nov 18 at 20:34




Thank you for your answer.
– David
Nov 18 at 20:34












Please have a look. math.stackexchange.com/questions/3003278/…
– David
Nov 18 at 20:35




Please have a look. math.stackexchange.com/questions/3003278/…
– David
Nov 18 at 20:35












Could you, please, support in order to calculate or approximate the sums represented in the above mentioned link?
– David
Nov 18 at 20:36




Could you, please, support in order to calculate or approximate the sums represented in the above mentioned link?
– David
Nov 18 at 20:36


















draft saved

draft discarded




















































Thanks for contributing an answer to Mathematics Stack Exchange!


  • Please be sure to answer the question. Provide details and share your research!

But avoid



  • Asking for help, clarification, or responding to other answers.

  • Making statements based on opinion; back them up with references or personal experience.


Use MathJax to format equations. MathJax reference.


To learn more, see our tips on writing great answers.





Some of your past answers have not been well-received, and you're in danger of being blocked from answering.


Please pay close attention to the following guidance:


  • Please be sure to answer the question. Provide details and share your research!

But avoid



  • Asking for help, clarification, or responding to other answers.

  • Making statements based on opinion; back them up with references or personal experience.


To learn more, see our tips on writing great answers.




draft saved


draft discarded














StackExchange.ready(
function () {
StackExchange.openid.initPostLogin('.new-post-login', 'https%3a%2f%2fmath.stackexchange.com%2fquestions%2f3002502%2fharmonic-series-partial-sum%23new-answer', 'question_page');
}
);

Post as a guest















Required, but never shown





















































Required, but never shown














Required, but never shown












Required, but never shown







Required, but never shown

































Required, but never shown














Required, but never shown












Required, but never shown







Required, but never shown







Popular posts from this blog

Plaza Victoria

In PowerPoint, is there a keyboard shortcut for bulleted / numbered list?

How to put 3 figures in Latex with 2 figures side by side and 1 below these side by side images but in...